Die Flugbahn eines Projektils, das von einem Hügel abgefeuert wird

Hier ist das Problem:

Ein Junge steht auf der Spitze eines Hügels, der gleichmäßig schräg abfällt ϕ . In welchem ​​Winkel θ Soll er einen Stein aus der Horizontalen werfen, damit er die größte Reichweite hat?

Mir ist klar, dass die gleiche Frage hier gepostet wird: https://physics.stackexchange.com/questions/24235/trajectory-of-projectile-thrown-downhill , aber ich habe einige Fragen, die in diesem Thread nicht beantwortet wurden:

  1. Lässt sich das Problem ohne Drehung des Koordinatensystems lösen? Wenn das so ist, wie?
  2. Ich habe versucht, das Problem mit einem gedrehten Koordinatensystem zu lösen, kann aber nicht herausfinden, wie ich es beenden kann (siehe die unten angegebene Arbeit).

Hier ist, was ich bisher habe:

  1. Wir richten das Koordinatensystem so ein, dass das Positive X Achse fällt mit dem Gefälle des Hügels zusammen. Dies vereinfacht das Problem, indem es uns ermöglicht, uns leicht zu beziehen ϕ Und θ , durch die Beziehung a = ϕ + θ .
  2. v 0 X = v 0 cos a
  3. v 0 j = v 0 Sünde a
  4. A X = G cos ( ϕ π 2 ) = G cos ( ( π 2 ϕ ) ) = G cos ( π 2 ϕ ) = G Sünde ϕ
  5. A j = G Sünde ( ϕ π 2 ) = G Sünde ( ( π 2 ϕ ) ) = G Sünde ( π 2 ϕ ) = G cos ϕ
  6. v X = v 0 X + 0 T A X ( T ' ) D T ' = v 0 cos a + 0 T ( G Sünde ϕ ) D T ' = v 0 cos a + T ( G Sünde ϕ )
  7. X = X 0 + 0 T v X ( T ' ) D T ' = 0 T ( v 0 cos a + T ' ( G Sünde ϕ ) ) D T ' = T ( v 0 cos a ) + 1 2 T 2 ( G Sünde ϕ )
  8. v j = v 0 j + 0 T A j ( T ' ) D T ' = v 0 Sünde a + 0 T ( G cos ϕ ) D T ' = v 0 Sünde a T ( G cos ϕ )
  9. j = j 0 + 0 T v j ( T ' ) D T ' = 0 T ( v 0 Sünde a T ' ( G cos ϕ ) ) D T ' = T ( v 0 Sünde a ) 1 2 T 2 ( G cos ϕ )
  10. Um die Flugzeit des Projektils zu finden, finden wir den Zeitpunkt, zu dem seine Flugbahn den Boden schneidet (in diesem Fall die X Achse), durch Einstellung j = 0 und Lösung für T .
    j = T ( v 0 Sünde a ) 1 2 T 2 ( G cos ϕ ) = 0
    v 0 Sünde a = 1 2 T ( G cos ϕ )
    T = 2 v 0 Sünde a G cos ϕ
  11. Ersetzen T in die Gleichung für X gibt uns die vom Projektil zurückgelegte Strecke als Funktion der Winkel an a Und ϕ .
    X = T ( v 0 cos a ) + 1 2 T 2 ( G Sünde ϕ )
    X = ( 2 v 0 Sünde a G cos ϕ ) ( v 0 cos a ) + 1 2 ( 2 v 0 Sünde a G cos ϕ ) 2 ( G Sünde ϕ )
    X = 2 v 0 2 G cos ϕ ( Sünde a cos a ) + 2 v 0 2 G cos ϕ ( Sünde 2 a Sünde ϕ cos ϕ )
    X = 2 v 0 2 G cos ϕ ( Sünde a cos a + Sünde 2 a bräunen ϕ )
  12. Mir ist aufgefallen, dass die Lösung im anderen Thread von hier aus durch Differenzieren vorgeht X gegenüber a , halten ϕ konstant, was gibt
    D X D a = 2 v 0 2 G cos ϕ ( D D a ( 1 2 ( Sünde ( 2 a ) + Sünde 2 a bräunen ϕ ) )
    D X D a = 2 v 0 2 G cos ϕ ( cos ( 2 a ) + 2 Sünde a cos a bräunen ϕ )
    D X D a = 2 v 0 2 G cos ϕ ( cos ( 2 a ) + Sünde ( 2 a ) bräunen ϕ )
    Diese Gleichung erlaubt uns zu untersuchen, wie X Änderungen bzgl a . Wir sehen das X steigt wie a nimmt bis zu einem bestimmten Punkt zu und nimmt dann wieder ab a steigt über diesen Wert. Dies bedeutet, dass der Graph von X hat ein relatives Maximum beim Wert von a was die maximale Reichweite bringt.
  13. Wir wollen den Wert von finden a das ergibt die maximale Reichweite des Geschosses. Mit anderen Worten, wir müssen den Wert von bestimmen a für die der Graph von X hat ein relatives Maximum. Dies erreichen wir durch Einstellung
    D X D a = 0 = 2 v 0 2 G cos ϕ ( cos ( 2 a ) + Sünde ( 2 a ) bräunen ϕ )
    Teilen Sie jede Seite durch 2 v 0 2 G cos ϕ produziert
    cos ( 2 a ) + Sünde ( 2 a ) bräunen ϕ = 0
    Hier verliere ich mich. Es scheint, als ob dies der einfache Teil sein sollte, denn das einzige, was noch zu tun ist, ist, die obige Gleichung zu lösen a , aber ich weiß nicht wie ich das machen soll. Kann mir bitte jemand diesen Teil erklären?

Außerdem würde ich gerne wissen, ob das Problem gelöst werden kann, ohne das Koordinatensystem zu drehen. Ich wollte es ursprünglich mit dem standardmäßigen rechtwinkligen Koordinatensystem lösen, verzettelte mich aber in einigen Gleichungen, die nirgendwohin zu führen schienen. Vielen Dank für Ihre Hilfe.

Kartesische Koordinaten: j , als positive Höhe, und X , als positiver Abstand, sind als Funktionen der Zeit gegeben. Sie können diese beiden durch die Linie gleichsetzen j = X bräunen ( ϕ ) . Dann wird der horizontale Bereich v 2 G ( Sünde ( 2 θ ) + bräunen ( ϕ ) cos ( 2 θ ) + bräunen ( ϕ ) ) was Sie nach der Maximierung irgendwo in die Nähe Ihres Ausgangsortes führt.

Antworten (1)

Ich bin nicht Ihre gesamte Arbeit durchgegangen, aber um die Frage zu beantworten, wie Sie diese Gleichung lösen können, ändern Sie bräunen ( ϕ ) Zu Sünde ( ϕ ) cos ( ϕ ) und nehme cos ( ϕ ) als gemeinsamen Nenner. Du erhältst

cos ( 2 a ) cos ( ϕ ) + Sünde ( 2 a ) Sünde ( ϕ ) cos ( ϕ ) = 0

Wenden Sie nun die Trig-Identität an cos ( A B ) = cos ( A ) cos ( B ) + Sünde ( A ) Sünde ( B ) und Ihre Gleichung reduziert sich auf

cos ( ϕ 2 a ) = 0
.